Difference between revisions of "2007 AMC 12A Problems/Problem 11"

(solution)
m (See also: - AMC 12 Box)
Line 23: Line 23:
  
 
== See also ==
 
== See also ==
{{AIME box|year=1984|num-b=6|num-a=8}}
+
{{AMC12 box|year=2007|ab=A|num-b=10|num-a=12}}
* [[AIME Problems and Solutions]]
 
* [[American Invitational Mathematics Examination]]
 
* [[Mathematics competition resources]]
 
  
 
[[Category:Intermediate Algebra Problems]]
 
[[Category:Intermediate Algebra Problems]]

Revision as of 02:31, 11 September 2007

Problem

The function f is defined on the set of integers and satisfies $f(n)= \begin{cases}  n-3 & \mbox{if }n\ge 1000 \\  f(f(n+5)) & \mbox{if }n<1000 \end{cases}$

Find $\displaystyle f(84)$.

Solution

Define $\displaystyle f^{h}(x) = f(f(\cdots f(f(x))\cdots))$, where the function $f$ is performed $h$ times. We find that $f(84) = f(f(89) = f^2(89) = f^3(94) = \ldots f^{y}(1004)$. $\displaystyle 1004 = 84 + 5(y - 1) \Longrightarrow y = 185$. So we now need to reduce $\displaystyle f^{185}(1004)$.

Let’s write out a couple more iterations of this function:

$\displaystyle f^{185}(1004) = f^{184}(1001) = f^{183}(998)$
$= f^{184}(1003) = f^{183}(1000) = f^{182}(997)$
$= f^{183}(1002) = f^{182}(999) = f^{183}(1004)$

So this function reiterates with a period of 2 for $x$. It might be tempting at first to assume that $f(1004) = 999$ is the answer; however, that is not true since the solution occurs slightly before that. Start at $f^3(1004)$:

$f^{3}(1004) = f^{2}(1001) = f(998)$
$= f^{2}(1003) = f(1000) = 997$

See also

2007 AMC 12A (ProblemsAnswer KeyResources)
Preceded by
Problem 10
Followed by
Problem 12
1 2 3 4 5 6 7 8 9 10 11 12 13 14 15 16 17 18 19 20 21 22 23 24 25
All AMC 12 Problems and Solutions